Prove that 0=1 in quantum mechanics

In summary, the commutator of two linear, hermitian operators with a value of i\hbar for the identity operator cannot have a normalized eigen vector solution. This is because the eigen problems of these operators do not have the same eigenvectors, and at least one of the operators must be unbounded. Therefore, the domain of physical observable A cannot be all of Hilbert space according to the Hellinger-Toeplitz theorem.
  • #1
Petar Mali
290
0
Of course every prove of this type have some mistake.

If nobody will see I will post solution!

[tex]\hat{A}[/tex], [tex]\hat{B}[/tex] - linear, hermitian operator which commutator is

[tex][\hat{A},\hat{B}]=i\hbar\hat{I}[/tex]

[tex]\hat{I}[/tex] - unit operator
Eigen problems of operators are:

[tex]\hat{A}|\psi \rangle=a|\psi \rangle[/tex]

[tex]\hat{B}|\psi \rangle=b|\psi \rangle[/tex]


[tex]|\psi \rangle[/tex] - normalized eigen vector

[tex]\langle \psi|\psi\rangle=1[/tex]

Look now

[tex][\hat{A},\hat{B}]=i\hbar\hat{I}[/tex]

Take a sandwich of right and left side with vector [tex]|\psi\rangle[/tex]

[tex]\langle \psi|[\hat{A},\hat{B}]|\psi\rangle=i\hbar\langle \psi|\hat{I}|\psi\rangle[/tex]

[tex]\frac{1}{i\hbar}\langle \psi|(\hat{A}\hat{B}-\hat{B}\hat{A})|\psi \rangle=1[/tex]

[tex]\frac{a}{i\hbar}(\langle \psi|\hat{B}|\psi\rangle-\langle \psi|\hat{B}|\psi\rangle)=1[/tex]

[tex]0=1[/tex]

Where is mistake? :D
 
Physics news on Phys.org
  • #2
If

[tex]A|a>=a|a>[/tex]

consider the state ket

[tex]|\psi>=e^{-ibB}|a>[/tex]

where b is a real number. Then

[tex]A|\psi>=Ae^{-ibB}|a>=\left(e^{-ibB}A+[A,e^{-ibB}]\right)|a> = \left(e^{-ibB}A-ib[A,B]e^{-ibB}\right)|a> = (a+b)e^{-ibB}|a>[/tex]

In other words

[tex]e^{-ibB}|a>=|a+b>[/tex]

and this is true for every [tex]a,b\in\mathbb{R}[/tex]. Thus A and B have continuous spectrum, and you can't normalize [tex]<a|a>=1[/tex]. Rather

[tex]<a|a'>=\delta(a-a')[/tex]

So you get something like

[tex]-ia<a|B|a'>=\delta(a-a')[/tex]

and this is not 1 = 0 when you put a = a'.
 
Last edited:
  • #3
No, it's far simpler than that. If [itex]A \lvert\psi\rangle = a \lvert\psi\rangle[/itex], and [itex]B \lvert\psi\rangle = b \lvert\psi\rangle[/itex], then
[tex][A,B] \lvert\psi\rangle = AB \lvert\psi\rangle - BA \lvert\psi\rangle = ab \lvert\psi\rangle - ba \lvert\psi\rangle = 0 \ne i\hbar I \lvert\psi\rangle.[/tex]
Hence [itex][A,B] \ne i\hbar I[/itex].
 
  • #4
adriank said:
No, it's far simpler than that. If [itex]A \lvert\psi\rangle = a \lvert\psi\rangle[/itex], and [itex]B \lvert\psi\rangle = b \lvert\psi\rangle[/itex], then
[tex][A,B] \lvert\psi\rangle = AB \lvert\psi\rangle - BA \lvert\psi\rangle = ab \lvert\psi\rangle - ba \lvert\psi\rangle = 0 \ne i\hbar I \lvert\psi\rangle.[/tex]
Hence [itex][A,B] \ne i\hbar I[/itex].

You simply proved that, since A and B do not commute, they cannot be simultaneously diagonalized. Petar Mali didn't use the fact that psi is also an eigenvector of B.
 
  • #5
If [tex]
[\hat{A},\hat{B}]=i\hbar\hat{I}
[/tex]

then [tex]
\langle \psi|\psi\rangle\ne1
[/tex]

because operators [tex]\hat{A}, \hat{B}[/tex] have continual spectrum!
 
  • #6
Yeah we solved the arcane! :biggrin:
 
  • #9
[tex]<A><B>\ne<AB>[/tex]
 
  • #10
Petar Mali said:
If [tex]
[\hat{A},\hat{B}]=i\hbar\hat{I}
[/tex]

then [tex]
\langle \psi|\psi\rangle\ne1
[/tex]

because operators [tex]\hat{A}, \hat{B}[/tex] have continual spectrum!

Do you know how to prove this claim?
 
  • #11
If

[tex]
A = \left(\begin{array}{cccc}
A_{11} & 0 & 0 & \cdots \\
0 & A_{22} & 0 & \cdots \\
0 & 0 & A_{33} & \cdots \\
\vdots & \vdots & \vdots & \ddots \\
\end{array}\right)
[/tex]

and

[tex]
B = \left(\begin{array}{cccc}
B_{11} & B_{12} & B_{13} & \cdots \\
B_{21} & B_{22} & B_{23} & \cdots \\
B_{31} & B_{32} & B_{33} & \cdots \\
\vdots & \vdots & \vdots & \ddots \\
\end{array}\right)
[/tex]

then

[tex]
AB-BA = \left(\begin{array}{cccc}
0 & B_{12}(A_{11} - A_{22}) & B_{13}(A_{11}-A_{33}) & \cdots \\
B_{21}(A_{22}-A_{11}) & 0 & B_{23}(A_{22}-A_{33}) & \cdots \\
B_{31}(A_{33}-A_{11}) & B_{32}(A_{33}-A_{22}) & 0 & \cdots \\
\vdots & \vdots & \vdots & \ddots \\
\end{array}\right)
[/tex]

so it is impossible to find [tex]A,B\in\mathbb{C}^{\mathbb{N}\times\mathbb{N}}[/tex] such that [itex]A[/itex] is diagonalizable and [itex]AB-BA=1[/itex].

On the other hand [itex]M_x(-\partial_x) - (-\partial_x)M_x = 1[/itex], where [itex]\partial_x[/itex] is the differentation operator and [itex]M_x[/itex] is the multiplication operator [itex](M_x\psi)(x)=x\psi(x)[/itex]. The claim about continuous spectrum and non-existence of (normalizable) eigenvectors seems to be correct.
 
  • #12
Petar Mali said:
[tex]\hat{A}|\psi \rangle=a|\psi \rangle[/tex]

[tex]\hat{B}|\psi \rangle=b|\psi \rangle[/tex]

The problem is not the continuous spectrum, although what you (and jostpuur) wrote is certainly right. The (bigger) problem is that two operators which don't commute don't have the same eigenvectors. The line above is impossible unless A and B commute. This whole exercise could be done with a discrete spectrum with a few minor changes. Or am I missing something?
 
  • #13
Fwiffo said:
Eigen problems of operators are:

[tex]\hat{A}|\psi \rangle=a|\psi \rangle[/tex]

[tex]\hat{B}|\psi \rangle=b|\psi \rangle[/tex]
The problem is not the continuous spectrum, although what you (and jostpuur) wrote is certainly right. The (bigger) problem is that two operators which don't commute don't have the same eigenvectors. The line above is impossible unless A and B commute. This whole exercise could be done with a discrete spectrum with a few minor changes. Or am I missing something?

Petar Mali didn't mean that the [itex]|\psi\rangle[/itex] would be the same on both of those lines.

Take a closer look at the steps leading to the contradiction:

Petar Mali said:
Take a sandwich of right and left side with vector [tex]|\psi\rangle[/tex]

[tex]\langle \psi|[\hat{A},\hat{B}]|\psi\rangle=i\hbar\langle \psi|\hat{I}|\psi\rangle[/tex]

[tex]\frac{1}{i\hbar}\langle \psi|(\hat{A}\hat{B}-\hat{B}\hat{A})|\psi \rangle=1[/tex]

[tex]\frac{a}{i\hbar}(\langle \psi|\hat{B}|\psi\rangle-\langle \psi|\hat{B}|\psi\rangle)=1[/tex]

[tex]0=1[/tex]

It is only assumed that [itex]A|\psi\rangle=a|\psi\rangle[/itex], and nothing about [itex]B[/itex].
 
  • #14
Petar Mali said:
If [tex]
[\hat{A},\hat{B}]=i\hbar\hat{I}
[/tex]

then [tex]
\langle \psi|\psi\rangle\ne1
[/tex]

because operators [tex]\hat{A}, \hat{B}[/tex] have continual spectrum!

jostpuur said:
Do you know how to prove this claim?

Here is a proof of something close to this.

Set [itex]\hbar = 1[/itex] and assume

[tex]AB - BA = iI.[/tex]

Multiplying the commutation relation by [itex]B[/itex] and reaaranging gives

[tex]
\begin{equation*}
\begin{split}
AB - BA &= iI \\
AB^2 - BAB &= iB \\
AB^2 - B \left( BA + iI \right) &= iB \\
AB^2 - B^2 A &= 2iB.
\end{split}
\end{equation*}
[/tex]

By induction,

[tex]AB^n - B^n A = niB^{n-1}[/tex]

for every positive integer [itex]n[/itex]. Consequently,

[tex]
\begin{equation*}
\begin{split}
n\left\| B \right\|^{n-1} &=\left\| AB^n - B^n A \right\| \\
&\leq 2\left\| A \right\| \left\| B \right\|^n\\
n &\leq 2\left\| A \right\| \left\| B \right\| .
\end{split}
\end{equation*}
[/tex]

Because this is true for every [itex]n[/itex], at least one of [itex]A[/itex] and [itex]B[/itex] must be unbounded. Say it is [itex]A[/itex]. Then, by the Hellinger-Toeplitz theorem, if [itex]A[/itex] is self-adjoint, the domain of physical observable [itex]A[/itex] cannot be all of Hilbert space!
 
  • #15
I think I pretty much answered my own question already. My calculation proves that if [itex]A[/itex] can be diagonalized so that it has eigenvectors in the Hilbert space, then [itex][A,B]\neq 1[/itex].

At this moment I'm not 100% sure that the non-existence of eigenvectors in the Hilbert space would be the same thing as the spectrum being continuous, though. Rigorous definitions should be recalled from somewhere...

George Jones, notice that you are proving a different thing. You proved unboundedness, but for example

[tex]
A = \left(\begin{array}{cccc}
1 & 0 & 0 & \cdots \\
0 & 2 & 0 & \cdots \\
0 & 0 & 3 & \cdots \\
\vdots & \vdots & \vdots & \ddots \\
\end{array}\right)
[/tex]

is diagonalized so that it has eigenvectors in the Hilbert space, despite the fact that it is unbounded.
 
  • #16
If you ignore the inconsistent and unnecessary assertion that [tex]|\psi\rangle[/tex] is an eigenvector of B, then there's no reason that [tex]|\psi\rangle[/tex] can't be normalizable. I think what the "proof" tells you, and what you'll find if you plug in some real operators with this property, is that the expression

[tex]\langle \psi|\hat{B}|\psi\rangle-\langle \psi|\hat{B}|\psi\rangle[/tex]

is an indeterminate expression like [tex]\infty - \infty[/tex] which can't be simplified down to 0.
 
  • #17
The_Duck said:
...there's no reason that [tex]|\psi\rangle[/tex] can't be normalizable...

I bet you can't find two hermitian operators with discrete spectrum, in any Hilbert space you like, with the commutation relation above...
 
  • #18
I repeat the original paradox with a "concrete" representation.

Operators are [itex]M_x[/itex] and [itex]-i\hbar\partial_x[/itex]. Then [itex][M_x,-i\hbar\partial_x]=i\hbar\;\textrm{id}[/itex].

We'll pick some eigenvector of [itex]M_x[/itex]. [itex]|\psi\rangle[/itex]! So that [itex]M_x|\psi\rangle = x_0|\psi\rangle[/itex] and [itex]\langle\psi|\psi\rangle=1[/itex]. In position representation this means

[tex]
\psi(x) = \frac{1}{\sqrt{\delta(0)}}\delta(x-x_0)
[/tex]

:wink:

Let's sandwich the commutator between brackets then!

[tex]
\langle\psi| [M_x,-i\hbar\partial_x] |\psi\rangle = \int\limits_{-\infty}^{\infty} \psi(x)^*
\big( M_x (-i\hbar\partial_x) - (-i\hbar\partial_x)M_x\big)\psi(x) dx
[/tex]
[tex]
= \frac{-i\hbar}{\delta(0)}\int\limits_{-\infty}^{\infty} \delta(x-x_0)\big(x \partial_x\delta(x-x_0)
-\partial_x(x\delta(x-x_0))\big) dx = \cdots
[/tex]

Now there is two different ways to continue. We can substitute

[tex]
\partial_x(x\delta(x-x_0)) = x\partial_x\delta(x-x_0) + \delta(x-x_0)
[/tex]

and then the calculation comes to end like this

[tex]
\cdots = \frac{-i\hbar}{\delta(0)}\int\limits_{-\infty}^{\infty}\delta(x-x_0)\big(-\delta(x-x_0)\big)dx = i\hbar
[/tex]

But we can also substitute

[tex]
x\delta(x-x_0) = x_0\delta(x-x_0)
[/tex]

Then the calculation comes to end like this

[tex]
\cdots = \frac{-i\hbar}{\delta(0)} x_0\int\limits_{-\infty}^{\infty} \delta(x-x_0)\big(\partial_x\delta(x-x_0)
-\partial_x\delta(x-x_0)\big)dx = 0
[/tex]

So [itex]i\hbar = 0[/itex]. Did it look rigorous? :biggrin:
 
  • #19
Petr Mugver said:
I bet you can't find two hermitian operators with discrete spectrum, in any Hilbert space you like, with the commutation relation above...

Ah; you're right; for some reason I thought that e.g. position eigenstates were normalizable in the desired way but of course they aren't.
 
  • #20
jostpuur said:
Petar Mali didn't mean that the [itex]|\psi\rangle[/itex] would be the same on both of those lines.

Take a closer look at the steps leading to the contradiction:



It is only assumed that [itex]A|\psi\rangle=a|\psi\rangle[/itex], and nothing about [itex]B[/itex].

Good point. This by the way is nice proof that the commutation relation [tex][A,B][/tex] [tex]= \alpha I[/tex] is impossible in the discrete case where operators can be diagonalized.
 
  • #21
The physical point about all this (see post above) is that the commutation relation we are talking about says that the one operator is the generator of translations for the other operator. Since you can translate eigenvalues by the continuum amount you like, theese eigenvalues must be... continuous!
 
  • #22
Petr Mugver said:
The physical point about all this (see post above) is that the commutation relation we are talking about says that the one operator is the generator of translations for the other operator. Since you can translate eigenvalues by the continuum amount you like, theese eigenvalues must be... continuous!

Good point!
This physical reason is elucidating.

This is pretty much similar to the trace paradox.
i.e. Using discrete basis and taking trace on both sides.
 
  • #23
jostpuur said:
[tex]
\psi(x) = \frac{1}{\sqrt{\delta(0)}}\delta(x-x_0)
[/tex]

[...]

Did it look rigorous?

Not even close. But you intended that post as a joke, right?
 
  • #24
strangerep said:
Not even close. But you intended that post as a joke, right?

Something like that. The intention was to repeat the same calculation that was done in the post #1. In my post #18 the calculation is done with delta functions and derivatives, so it is easy to see that the calculation is uncertain. In the post #1 it is done with brackets, so the calculation was camouflaged as rigorous.
 
  • #25
There's one more point of view that can help bringing clarity to the paradox. If we approximate the real line as a discrete, by choosing some [itex]\Delta x[/itex] to denote the smallest step on the real line, then multiplication and differentiation operators are

[tex]
M_x = \Delta x\left(\begin{array}{ccc|cccc}
\ddots & \vdots & \vdots & \vdots & \vdots & \vdots & \\
\cdots & -2 & 0 & 0 & 0 & 0 & \cdots \\
\cdots & 0 & -1 & 0 & 0 & 0 & \cdots \\
\hline
\cdots & 0 & 0 & 0 & 0 & 0 & \cdots \\
\cdots & 0 & 0 & 0 & 1 & 0 & \cdots \\
\cdots & 0 & 0 & 0 & 0 & 2 & \cdots \\
& \vdots & \vdots & \vdots & \vdots & \vdots & \ddots \\
\end{array}\right)
[/tex]

[tex]
\partial_x = \frac{1}{\Delta x} \left(\begin{array}{ccc|cccc}
\ddots & \vdots & \vdots & \vdots & \vdots & \vdots & \\
\cdots & -1 & 1 & 0 & 0 & 0 & \cdots \\
\cdots & 0 & -1 & 1 & 0 & 0 & \cdots \\
\hline
\cdots & 0 & 0 & -1 & 1 & 0 & \cdots \\
\cdots & 0 & 0 & 0 & -1 & 1 & \cdots \\
\cdots & 0 & 0 & 0 & 0 & -1 & \cdots \\
& \vdots & \vdots & \vdots & \vdots & \vdots & \ddots \\
\end{array}\right)
[/tex]

The commutator looks like this

[tex]
M_x\partial_x - \partial_x M_x = \left(\begin{array}{cc|cccc}
\ddots & \vdots & \vdots & \vdots & \vdots & \\
\cdots & 0 & -1 & 0 & 0 & \cdots \\
\hline
\cdots & 0 & 0 & -1 & 0 & \cdots \\
\cdots & 0 & 0 & 0 & -1 & \cdots \\
\cdots & 0 & 0 & 0 & 0 & \cdots \\
& \vdots & \vdots & \vdots & \vdots & \ddots \\
\end{array}\right)
[/tex]

That means that

[tex]
[M_x,-i\hbar\partial_x] \approx i\hbar\;\textrm{id}
[/tex]

only approximately. If we take an eigenvector [itex]|\psi\rangle[/itex] such that [itex]M_x|\psi\rangle=x_0|\psi\rangle[/itex], then

[tex]
\langle\psi|[M_x,-i\hbar\partial_x]|\psi\rangle = 0
[/tex]

is the right answer, because the commutator will translate [itex]|\psi\rangle[/itex] such that it becomes an eingenstate with eigenvalue [itex]x_0-\Delta x[/itex]. The paradox is related to the orthogonality of the eigenstates corresponding to delta spikes at [itex]x_0[/itex] and [itex]x_0-\Delta x[/itex], when [itex]\Delta x[/itex] is considered as zero.
 
  • #26
George Jones said:
[tex]AB - BA = iI.[/tex]

Multiplying the commutation relation by [itex]B[/itex] and reaaranging gives

[tex]
\begin{equation*}
\begin{split}
AB - BA &= iI \\
AB^2 - BAB &= iB \\
AB^2 - B \left( BA + iI \right) &= iB \\
AB^2 - B^2 A &= 2iB.
\end{split}
\end{equation*}
[/tex]

By induction,

[tex]AB^n - B^n A = niB^{n-1}[/tex]

for every positive integer [itex]n[/itex].

This can be applied to the following identity too.

[tex]
[A,e^{-ibB}] = \sum_{n=0}^{\infty} \frac{(-ib)^n}{n!}(AB^n - B^nA) = \sum_{n=1}^{\infty}\frac{(-ib)^n}{n!}niB^{n-1} = be^{-ibB}
[/tex]

Suddenly Peter Mugver's earlier post started to make sense.

Petr Mugver said:
[tex]A|\psi>=Ae^{-ibB}|a>=\left(e^{-ibB}A+[A,e^{-ibB}]\right)|a> = \left(e^{-ibB}A-ib[A,B]e^{-ibB}\right)|a> = (a+b)e^{-ibB}|a>[/tex]

In other words

[tex]e^{-ibB}|a>=|a+b>[/tex]

and this is true for every [tex]a,b\in\mathbb{R}[/tex]. Thus A and B have continuous spectrum, and you can't normalize [tex]<a|a>=1[/tex]. Rather

[tex]<a|a'>=\delta(a-a')[/tex]

I don't see what was the point of an intermediate step [itex]-ib[A,B]e^{-ibB}[/itex], but it's right anyway, I see.
 
  • #27
jostpuur said:
I don't see what was the point of an intermediate step [itex]-ib[A,B]e^{-ibB}[/itex], but it's right anyway, I see.

It's a property of Commutators:

[tex][A,f(B)]=[A,B]f'(B)[/tex]

[tex][f(A),B]=f'(A)[A,B][/tex]
 
  • #28
Petr Mugver said:
It's a property of Commutators:

[tex][A,f(B)]=[A,B]f'(B)[/tex]

[tex][f(A),B]=f'(A)[A,B][/tex]

It's not actually a property of all commutators, but only if
[A,B] commutes with B (which is of course the case for the
Heisenberg algebra).
 
Last edited:
  • #29
strangerep said:
It's not actually a property of any commutators, but only if
[A,B] commutes with B (which is of course the case for the
Heisenberg algebra).

Yeah it's true... forgive me! :rolleyes:
 

1. Is it possible for 0 to equal 1 in quantum mechanics?

No, it is not possible for 0 to equal 1 in quantum mechanics. This would violate the fundamental principles and equations of quantum mechanics.

2. Can you provide a mathematical proof that 0=1 in quantum mechanics?

No, there is no valid mathematical proof that 0 can equal 1 in quantum mechanics. Mathematical equations and principles in quantum mechanics do not allow for such an equation to hold true.

3. Are there any experiments or observations that support the idea of 0=1 in quantum mechanics?

No, there are no experiments or observations that support the idea of 0=1 in quantum mechanics. On the contrary, all experiments and observations in quantum mechanics have consistently shown that 0 does not equal 1.

4. Why is it important to disprove the idea of 0=1 in quantum mechanics?

It is important to disprove the idea of 0=1 in quantum mechanics because it goes against the fundamental principles and equations of this field of science. Accepting 0=1 would lead to inconsistencies and contradictions in our understanding of the universe.

5. Are there any theoretical scenarios or models that allow for 0=1 in quantum mechanics?

No, there are no theoretical scenarios or models that can accommodate the idea of 0=1 in quantum mechanics. All current models and theories in quantum mechanics are based on the fundamental principle that 0 does not equal 1.

Similar threads

  • Quantum Physics
Replies
4
Views
757
Replies
10
Views
1K
Replies
11
Views
1K
  • Quantum Physics
Replies
13
Views
1K
Replies
2
Views
566
Replies
2
Views
310
  • Quantum Physics
Replies
1
Views
681
Replies
2
Views
1K
  • Quantum Physics
Replies
9
Views
859
Back
Top